0 of 15 Questions completed
Questions:
You have already completed the quiz before. Hence you can not start it again.
Quiz is loading…
You must sign in or sign up to start the quiz.
You must first complete the following:
0 of 15 Questions answered correctly
Your time:
Time has elapsed
You have reached 0 of 0 point(s), (0)
Earned Point(s): 0 of 0, (0)
0 Essay(s) Pending (Possible Point(s): 0)
Average score |
|
Your score |
|
Deep brain stimulation has been FDA approved for the management of medically refractory epilepsy when it targets which of the following structures?
A 65-year-old man presents to the emergency room with progressive weakness over the course of several months. Examination shows pure motor weakness with no sensory involvement, decreased reflexes, and diffuse fasciculations. An EMG shows conduction block of motor neurons outside of compression sites and normal sensory conduction velocities. Which of the following antibodies is associated with this disease?
Which of the following is the most common gene associated with early-onset Alzheimer’s disease?
A 35-year-old female with a history of heroin abuse was found unresponsive and received 35 minutes of CPR before a return of spontaneous circulation (ROSC). She was intubated in the field and was brought urgently to the hospital. While in the ICU, the nursing staff noticed abnormal movements concerning for possible seizure. An EEG was performed and shown below. What is this finding most consistent with?
A 62-year-old 50 kg female with a history of myasthenia gravis presents to the hospital with shortness of breath and generalized weakness felt to be secondary to a myasthenic crisis. Her medication history in the chart states she is currently taking pyridostigmine 60 mg QID, prednisone 20 mg daily, and just started cyclophosphamide 4 weeks ago. She was safely intubated in the ED with a NIF of -18 cm H2O and a VC of 600 ml. Select the appropriate next steps regarding pharmacotherapy and management for a myasthenic crisis.
I. Discontinue pyridostigmine
II. Increase the dose of prednisone
III. Increase the dose of azathioprine
IV. Consult general surgery for thymectomy
V. Consider PLEX or IVIg therapy
VI. Start rituximab
Which of the following is present in the image shown below?
A 54-year-old male with hypertension, hyperlipidemia, and atrial fibrillation on warfarin is newly diagnosed with epilepsy. Of the following medications, which is the most appropriate anti-seizure medication to prescribe?
Which of the following genes is responsible for familial ALS?
A 49-year-old woman with recurrent episodes of vertigo and has an MRI brain performed at the request of her neurologist. Imaging reveals a well-circumscribed dural-based homogenous contrast-enhancing lesion. What is the most likely etiology of the lesion outlined above?
Paralysis of the stapedius muscle can occur with ipsilateral damage to which cranial nerve?
What type of tumor is showcased in the pathologic image below?
A 78-year-old male with a past medical history of diabetes presents to the clinic with a 2-year history of progressive lower extremity numbness and pain. He describes the pain as “pins and needles”. Physical examination reveals loss of pinprick sensation to the ankles bilaterally, mute Achilles reflexes, and moderate difficulty with tandem gait. A diagnosis is made based on examination and clinical history. What is the mechanism of action of the most appropriate first-line therapeutic agent?
A 35-year-old male presents to the neurology clinic for the management of seizures. The patient reports having focal to bilateral tonic-clonic seizures (FTBTC) since the age of 15. His seizures are usually preceded by the smell of burning rubber. He will then have left hand automatisms followed by whole-body tonic-clonic activity. Levetiracetam monotherapy was initially effective. However, 1 year ago seizures returned, occurring once every two months. Lacosamide was added 6 months ago but there has been no significant improvement in his seizure frequency. Levels of these two anti-seizure medications (ASMs) are therapeutic and there have been no concerns regarding medication adherence. What is the most appropriate next step in management?
A 34-year-old man with medically-refractory epilepsy has an MRI brain completed. The lesion shown (T2 sequence) does not enhance with contrast. If a surgical resection was performed, which of the following would be the most likely seen on pathologic analysis?
What is the state of the patient in the EEG shown below?